LSAT and Law School Admissions Forum

Get expert LSAT preparation and law school admissions advice from PowerScore Test Preparation.

 Administrator
PowerScore Staff
  • PowerScore Staff
  • Posts: 8917
  • Joined: Feb 02, 2011
|
#36387
Complete Question Explanation

Main Point. The correct answer choice is (D)

The stimulus opens with a presentation of competing viewpoints on Mayor McKinney. The Mayor’s
policies have been criticized as benefi ting only the wealthy residents of the city, but the author states
that this criticism is unfair, and this is the conclusion of the argument. In the remaining sentences the
author offers supporting evidence for that conclusion, namely that McKinney’s policies benefi t all by
keeping housing costs in check.

Remember that in Main Point questions you may encounter one or more answer choices that
express statements supported by the stimulus, but you are required to select the statement that is the
conclusion.

Answer choice (A): The stimulus did not attempt to decide which group McKinney favors. We know
that the critics believe that McKinney favors the wealthy, but the author’s exact position is unknown
(only that some of McKinney’s policies favor the less affl uent), and certainly the main point is not
that it is impossible to tell which group McKinney favors.

Answer choice (B): This answer choice refers to the position of McKinney’s critics, and that is a
position that the author rebuts. This response is exactly opposite of what is required.

Answer choice (C): This is a premise of the argument, not the conclusion. The premise of an
argument is never the correct answer in a Main Point question.

Answer choice (D): This is the correct answer choice. This answer paraphrases the author’s stance
in the fi rst sentence. According to the argument, the criticisms are unfair, and incorrect, so they are in
fact unjustifi ed.

Answer choice (E): This is a premise in the argument, and supports the idea that McKinney’s
policies have benefi ted the less affl uent. This response is incorrect because the correct answer should
refl ect the conclusion of the argument.

Get the most out of your LSAT Prep Plus subscription.

Analyze and track your performance with our Testing and Analytics Package.